Difference between revisions of "2016 AMC 12B Problems/Problem 15"

m (Solution 3 (really fast))
m (Solution 4)
 
(6 intermediate revisions by 3 users not shown)
Line 13: Line 13:
  
 
==Solution 2==
 
==Solution 2==
We proceed from the factorization in the above solution. By the AM-GM inequality,
+
We'll proceed from the factoring process above.
 +
By the AM-GM inequality,
  
 
<cmath>\frac{a_1+a_2+a_3}{3}\geq\sqrt[3]{a_1a_2a_3}</cmath>
 
<cmath>\frac{a_1+a_2+a_3}{3}\geq\sqrt[3]{a_1a_2a_3}</cmath>
Line 33: Line 34:
 
==Solution 3 (really fast)==
 
==Solution 3 (really fast)==
 
First, we see that we want to try and maximize each vertex. Since the multiplication of each vertex is the product of three values, we want to maximize those three values. Doing so, we see that we want them to be as close as possible, giving <math>4.5^3</math> (the average of all the values). This gives us the maximum for each vertex, multiplied by the 8 vertices, yields our answer <math>\boxed{\textbf{(D)}\ 729 }</math> Also note that if you cannot evaluate <math>4.5^3</math> quickly, a rough approximation of <math>5*4.5*4*8</math> will yield 720, very close to our answer. -rayprati
 
First, we see that we want to try and maximize each vertex. Since the multiplication of each vertex is the product of three values, we want to maximize those three values. Doing so, we see that we want them to be as close as possible, giving <math>4.5^3</math> (the average of all the values). This gives us the maximum for each vertex, multiplied by the 8 vertices, yields our answer <math>\boxed{\textbf{(D)}\ 729 }</math> Also note that if you cannot evaluate <math>4.5^3</math> quickly, a rough approximation of <math>5*4.5*4*8</math> will yield 720, very close to our answer. -rayprati
 +
 +
 +
Note about note above: <math>4.5=\frac{9}{2}.</math> So, we want to evaluate <math>(\frac{9}{2})^3 \cdot 8 = \frac{729}{8} \cdot 8 = 729.</math> So, it’s quite easy to evaluate ~solasky
 +
 +
==Solution 4==
 +
 +
It is obvious to put <math>5</math>, <math>6</math>, and <math>7</math> on the faces that share the same vertex. As <math>4</math> is the next biggest number, the face with <math>4</math> has to be next to the faces with <math>6</math> and <math>7</math>. As <math>4</math> is the next biggest number, the face with <math>3</math> has to be next to the faces with <math>5</math> and <math>7</math>. making the face with <math>2</math> next to the faces with <math>6</math> and <math>5</math>.
 +
 +
Therefore the answer is <math>7 \cdot 5 \cdot 6 + 7 \cdot 4 \cdot 6 + 7 \cdot 3 \cdot 5 + 7 \cdot  3 \cdot 4 + 2 \cdot 5 \cdot 6 + 2 \cdot 4 \cdot 6 + 2 \cdot 3 \cdot 5 + 2 \cdot  3 \cdot 4 = \boxed{\textbf{(D)}\ 729 }</math>
 +
 +
Note: 1680 is impossible to achieve as it requires <math>210 = 7 \cdot 6 \cdot 5</math> for each vertex.
 +
 +
~[https://artofproblemsolving.com/wiki/index.php/User:Isabelchen isabelchen]
  
 
==See Also==
 
==See Also==
 
{{AMC12 box|year=2016|ab=B|num-b=14|num-a=16}}
 
{{AMC12 box|year=2016|ab=B|num-b=14|num-a=16}}
 
{{MAA Notice}}
 
{{MAA Notice}}

Latest revision as of 01:25, 4 August 2023

Problem

All the numbers $2, 3, 4, 5, 6, 7$ are assigned to the six faces of a cube, one number to each face. For each of the eight vertices of the cube, a product of three numbers is computed, where the three numbers are the numbers assigned to the three faces that include that vertex. What is the greatest possible value of the sum of these eight products?

$\textbf{(A)}\ 312 \qquad \textbf{(B)}\ 343 \qquad \textbf{(C)}\ 625 \qquad \textbf{(D)}\ 729 \qquad \textbf{(E)}\ 1680$

Solution 1

First assign each face the letters $a,b,c,d,e,f$. The sum of the product of the faces is $abc+acd+ade+aeb+fbc+fcd+fde+feb$. We can factor this into $(a+f)(b+c)(d+e)$ which is the product of the sum of each pair of opposite faces. In order to maximize $(a+f)(b+c)(d+e)$ we use the numbers $(7+2)(6+3)(5+4)$ or $\boxed{\textbf{(D)}\ 729 }$.

Solution 2

We'll proceed from the factoring process above. By the AM-GM inequality,

\[\frac{a_1+a_2+a_3}{3}\geq\sqrt[3]{a_1a_2a_3}\]

Cubing both sides,

\[\left(\frac{a_1+a_2+a_3}{3}\right)^3\geq{a_1a_2a_3}\]

Let $a_1=(a+f)$, $a_2=(b+c)$, and $a_3=(d+e)$. Let's substitute in these values.

\[\left(\frac{a+b+c+d+e+f}{3}\right)^3\geq{(a+f)(b+c)(d+e)}\]

$a+b+c+d+e+f$ is fixed at 27.

\[\left(\frac{27}{3}\right)^3\geq{(a+f)(b+c)(d+e)}\]

\[\boxed{\textbf{(D)}\ 729 }\geq{(a+f)(b+c)(d+e)}\]

Solution 3 (really fast)

First, we see that we want to try and maximize each vertex. Since the multiplication of each vertex is the product of three values, we want to maximize those three values. Doing so, we see that we want them to be as close as possible, giving $4.5^3$ (the average of all the values). This gives us the maximum for each vertex, multiplied by the 8 vertices, yields our answer $\boxed{\textbf{(D)}\ 729 }$ Also note that if you cannot evaluate $4.5^3$ quickly, a rough approximation of $5*4.5*4*8$ will yield 720, very close to our answer. -rayprati


Note about note above: $4.5=\frac{9}{2}.$ So, we want to evaluate $(\frac{9}{2})^3 \cdot 8 = \frac{729}{8} \cdot 8 = 729.$ So, it’s quite easy to evaluate ~solasky

Solution 4

It is obvious to put $5$, $6$, and $7$ on the faces that share the same vertex. As $4$ is the next biggest number, the face with $4$ has to be next to the faces with $6$ and $7$. As $4$ is the next biggest number, the face with $3$ has to be next to the faces with $5$ and $7$. making the face with $2$ next to the faces with $6$ and $5$.

Therefore the answer is $7 \cdot 5 \cdot 6 + 7 \cdot 4 \cdot 6 + 7 \cdot 3 \cdot 5 + 7 \cdot  3 \cdot 4 + 2 \cdot 5 \cdot 6 + 2 \cdot 4 \cdot 6 + 2 \cdot 3 \cdot 5 + 2 \cdot  3 \cdot 4 = \boxed{\textbf{(D)}\ 729 }$

Note: 1680 is impossible to achieve as it requires $210 = 7 \cdot 6 \cdot 5$ for each vertex.

~isabelchen

See Also

2016 AMC 12B (ProblemsAnswer KeyResources)
Preceded by
Problem 14
Followed by
Problem 16
1 2 3 4 5 6 7 8 9 10 11 12 13 14 15 16 17 18 19 20 21 22 23 24 25
All AMC 12 Problems and Solutions

The problems on this page are copyrighted by the Mathematical Association of America's American Mathematics Competitions. AMC logo.png